Hey Everyone,
Got this question right when I did this initially, but having a tough time ruling out answer (D). I know at first glance it seems out of scope, but here's my reasoning why (D) could be right. If all scientific investigators don't ...
I still don't understand why answer choice E is wrong. My understanding is, iIf stress is a symptom of a weakened immune system, then wouldn't that mean that symptoms such as stress then lead to or cause the weakened immune system?
This question really baffles me. How can B be correct? If the probe reveals that the light reflected is much less, how would that support B? Please help, I'm so confused!
In answer B, he says she says "Hey, recycled paper isn't necessarily lower quality. In fact, the best paper was made was made from recycled materials..."
Hey guys, I'm doing some older questions as a 5th section on my PTs, and I decided to take PT 7's first LR section. I'm BRing it right now, and I can't for the life of me figure this one out; I skipped it twice during the exam, and I'm still just as ...
The question stem in this question reads: "In order for the conclusion that Bevex is safe for people to be properly drawn, which one of the following must be true?"
Before doing blind review, I labelled this question as a MBT question (as is also ...
The correct answer choice (E) states that the argument "implies that brains and muscle are similar in one respect because they are similar in another respect." If the first respect refers to being able to improve if exercised, what is the latter respect?
Can anyone please tell me how the answer choice (B) is correct and (C) is wrong?? I just can't get myself to understand. This just sounds like the question is playing word games.
Why not D? if (D) wasn't true and not all scientific investigators received a grant, then wouldn't that mean serendipity can play a role in scientific discovery? Since they would not have to make clear projections and disregard info not directly related to ...
Can someone please help me comprehend why "reducing stress can help weakened immune system" is same as "stress can weaken the immune system" ? It seems like invalid negation to me...
#help
Question 10: How do we know that "additives" was ever the cause of behavioral problems in the first place? Researchers being "trained to assess the presence or absence of behavioral problems" does not mean whatever behavioral problems are ...
Could someone explain why the correct answer choice is E ? Also is there a way for me to look up an explanation for this specific question on 7sage? I just started the free trial so I am still getting used to the site?
yhe right answer is E and i picked B. I dont get at all why E is right because isnt that going against our whole conclusion which is that what the Marine Biologists are saying (lobsters eat one another is in response to hunger when they are together) is ...
Hi everyone, just want to confirm the validity of the answer choices. I find this question confusing, even though I can see (A) is the best AC.
The breakdown of the core is:
**ignore anything not directly related to the funded research ——> ...
The answer for this question is B. I understand why B is right, but I was not able to fully eliminate why D is wrong (I understand why A, C, and E are wrong). The reason why I am still not sure why D is wrong is because D states that, ...
In this passage, I picked D because I interpreted Rito using the phrase "serious underestimates" as the severity of the underestimate of the raw numbers, while Hiro uses “surely underestimate” as an acceptance that there is an underestimate but not ...
I would love to Blind Review with someone - anyone! I am starting off my LR journey and it could be great to bounce ideas off with someone else. My LR scores are not very consistent yet and can range from timed -9/-3 and BR -5/-2. I am trying to make my ...
Could someone help me ID what flaw is in the stimulus and why answer choice D is wrong? Is D incorrect because the skeptic is assuming Debbie used another technique aside from the three he was originally testing for? And is the sleight of hand, trick deck, ...
The title is pretty self-explanatory but I need help on #7. Why is E incorrect? And what makes A the right fit? I feel that I have no strategy when it comes to these argument completion style questions, especially when it comes to answer choices that just ...